Matrix of Transformation (non standard basis)

Click For Summary
SUMMARY

The discussion centers on the linear transformation T defined from R3 to R3 by T(x1, x2, x3) = (x1, x1 + x2, x1 + x2 + x3). The matrix representation [T] relative to the standard basis is confirmed as [[1, 0, 0], [1, 1, 0], [1, 1, 1]]. The transformation matrix [T]' relative to the basis B' = {(1,0,0)T, (1,1,0)T, (1,1,1)T} was initially miscalculated but corrections were provided. The transition matrix Q from the standard basis to B' was also derived, with an emphasis on ensuring the accuracy of the inverse matrix calculations.

PREREQUISITES
  • Understanding of linear transformations and their properties
  • Familiarity with matrix representation of transformations
  • Knowledge of basis transformations in vector spaces
  • Ability to compute matrix inverses
NEXT STEPS
  • Study the properties of linear transformations in depth
  • Learn how to derive transition matrices between different bases
  • Practice calculating matrix inverses and their applications
  • Explore examples of linear transformations in higher dimensions
USEFUL FOR

Students and educators in linear algebra, mathematicians working with vector spaces, and anyone involved in computational mathematics or applied linear transformations.

margaret37
Messages
12
Reaction score
0

Homework Statement



Define T : R3x1 to R3x1 by T = (x1, x2,x3)T = (x1, x1+x2, x1+x2+x3)T

1 Show that T is a linear transformation
2 Find [T] the matrix of T relative to the standard basis.
3 Find the matrix [T]' relative to the basis
B' = {(1,0,0)t, (1,1,0)t, (1,1,1)t
4 Find the transition matrix Q from B to B'
5 Verify that [T]' = Q[T]Q-1



Homework Equations





The Attempt at a Solution


1) I think this is done I showed that T(c(alpha + beta)) = c* T(alpha) + T(Beta) by substituting in vectors (a1, a2, a3) and (b1,b2,b3) and working through the algebra.

2) This is what I did, I think I understood this properly:

I built a transformation Matrix [T] = T(e1) | T(e2) | T(e3)

I got
1 0 0
1 1 0
1 1 1

3) Then I think is wrong but I'm not sure

I built another matrix [T]' = T(v1) | T(v2) | t(v3)
where v1, v2, and v3 are the the vectors given above

I got
1 1 1
1 2 2
2 2 3


4) Then I built a transformation matrix by finding the coordinates of e1, e2, and e3 relative to v1, v2 and v3

I got
1 -1 0
0 1 -1
0 0 1

I calculated an inverse

1 1 0
0 1 1
0 0 1

5) Then if I had done everything right
[T]' should have been equal to Q[T]Q-1

But it wasn't even close

1 1 1
0 1 1
0 0 1

Any help would be greatly appreciated.
Margaret
 
Physics news on Phys.org
3) Then I think is wrong but I'm not sure

I built another matrix [T]' = T(v1) | T(v2) | t(v3)
where v1, v2, and v3 are the the vectors given above

I got
1 1 1
1 2 2
2 2 3


One error is that T(v1) should be (1,1,1)t instead of (1,1,2)t. A more important error is that the resulting numbers "1,1,1" are coefficients with respect to the standard basis, not B'. You need to figure out x, y, and z such that
(1,1,1)t=x v1 + y v2 + z v3. This is easy to do by inspection: x=0, y=0, z=1. The first column of [T]' is therefore (0,0,1)t. Do a similar computation for T(v2) and T(v3). The appropriate coefficients x, y, and z are easy to find by inspection, although not as trivial as for T(v1).

Also, the inverse of

1 -1 0
0 1 -1
0 0 1

is almost but not quite

1 1 0
0 1 1
0 0 1

The row 1 column 3 entry is incorrect.
 
Thank you very much. I think I've got it now.
 
Question: A clock's minute hand has length 4 and its hour hand has length 3. What is the distance between the tips at the moment when it is increasing most rapidly?(Putnam Exam Question) Answer: Making assumption that both the hands moves at constant angular velocities, the answer is ## \sqrt{7} .## But don't you think this assumption is somewhat doubtful and wrong?

Similar threads

  • · Replies 6 ·
Replies
6
Views
1K
Replies
8
Views
3K
  • · Replies 1 ·
Replies
1
Views
2K
  • · Replies 10 ·
Replies
10
Views
3K
  • · Replies 3 ·
Replies
3
Views
2K
  • · Replies 4 ·
Replies
4
Views
1K
Replies
3
Views
2K
Replies
4
Views
2K
  • · Replies 2 ·
Replies
2
Views
2K
  • · Replies 1 ·
Replies
1
Views
2K